If the strawberry layer is not the top layer, then which one of the following is a complete and accurate list of the ...

Anna20 on July 20, 2020

Timing

What is the best approach for this question in your view? I find I am spending a lot of time here. I looked at the answer choices, and made a note that the first hypothetical shows V in 5th. Then I tried to think of set ups, where V came in fourth. I tried LMOVS (but you can't have R next to S) and then RLMVO (but S can't be 6th in this question). So then concluded the answer must be E. It feels like quite a lot of time is spent on trial and error here - trying to figure out whether V would work in the 4th position.

Reply
Create a free account to read and take part in forum discussions.

Already have an account? log in

shunhe on July 21, 2020

Hi @Anna2020,

Thanks for the question! So another way to approach this question is to think: what are the consequences of S not being the top, or sixth layer? What must follow from that? Making S not be in the top layer should naturally constrict where V can go, which might help us narrow down our answer choices here.

So let’s say that S isn’t sixth. Well, We know that O, M, and L all have to follow S based on the rules. So S also can never be 1st, 2nd, or 3rd layer. That means there’s two places where we can put S: we can put S in the fourth layer, or we can put S in the fifth layer.

Let’s put S in the fourth layer first. Well, we know that O is third, M is 2nd, and L is in the first layer. We know that R isn’t next to S, so V has to be in fifth, and R is in sixth. So when S is in the fourth layer, we have one possibility:

LMOSVR

OK, well obviously V can be in fifth, so that didn’t eliminate anything. But now let’s say we put S in fifth. That means LM has to be in layers 1 and 2 or in layers 2 and 3.

Let’s first say that LM are in layers 2 and 3, since that forces O to be in layer 4. Well, R can’t be next to S, so R has to be in layer 1, and V has to be in layer 6. So again, there’s only one possibility, which is

RLMOSV

Now let’s say that LM are in layers 1 and 2. Well, R can’t be next to S, so R has to be in layer 3. And O has to be in layer 4. That means that V again has to be in layer 6; again, there’s only one possibility, which is

LMROSV

And we can see that when S isn’t sixth, there’s only three possibilities for the game. And in these three possibilities, V is only in fifth or sixth. So (E) is going to be the correct answer here. This approach has you look at the consequences of the hypothetical and work from there, and probably takes less time.

Hope this helps! Feel free to ask any other questions that you might have.